0
$\begingroup$

[Cross-posted from MS after 8 days without reply.] I have a real matrix $R_{ij} \in \mathbb{R}^{n \times m}$ whose entries are sampled iid from an absolutely continuous distribution $D$; a fixed real 3-dimensional tensor $T_{ijk} \in \mathbb{R}^{n \times n \times m}$; and any matrix $A_{ij}$ which satisfies $$ A_{ij} = R_{ij} + \sum_k T_{ijk} A_{ik} \,. $$ I would like to show that $A_{ij}$ is almost surely full-rank. I know that the set of matrices with rank-deficient matrices has zero measure (so $R_{ij}$ is almost surely full-rank), and tried to use the fact that $rank(AA^T) = rank(A^TA) = rank(A)$, or to re-express $$ \sum_k A_{ik}(\delta_{jk} - T_{ijk}) = R_{ij} \,, $$ to no avail. In the case of $n = m$, I was hoping to take the determinant on both sides and conclude, but don't know how to take the determinant because of the 3D tensor on the LHS. Any ideas?

Edit: if I moreover assume that the matrix $S_i = T_{ijk}$, for each fixed $i$, has a spectrum bounded by 1, we can expand the equation to get $$ A_{ij} = \sum_k R_{ik}\sum_{n = 0}^{\infty} (S_i^n)_{ijk} = \sum_k R_{ik}(I-S_i)^{-1}_{ijk} \,, $$ from which we conclude that the entries of $A_{ij}$ are also absolutely continuous and independent, and thus has full-rank with probability 1. Is this correct? Can we somehow generalise this without assuming a bounded spectrum?

$\endgroup$
4
  • $\begingroup$ In which sense this is a recurrence? What is a time variable? Looks like just a system of equations. $\endgroup$ Commented Jul 10 at 22:27
  • $\begingroup$ In addition to Fedor Petrov's comment: Why are you saying that $A$ is defined by that matrix equation that you refer to as "the recurrence relation"? The matrix equation is a system of $nm$ scalar linear equations with $nm$ unknowns. The determinant of such a system may be $0$, and then the system may have no or infinitely many solutions (actually, no solutions with probability $1$). $\endgroup$ Commented Jul 11 at 3:01
  • $\begingroup$ Apologies, the word recurrence was used incorrectly; I removed it everywhere! $\endgroup$ Commented Jul 11 at 7:02
  • $\begingroup$ Couldn't you just scale the equation such that $S_i = R_{ijk}$ has spectrum bounded by $1$. Since the distribution of the entries of $R$ are absolutely continuous, the distribution might not be scaling invariant, but the property that $R$ is of full rank with probability $1$ might be preserved. $\endgroup$ Commented Jul 13 at 17:40

1 Answer 1

4
$\begingroup$

So you have a linear map $\tau$ from ${\rm Mat}(m,n)$ to itself and $A$ is defined by $\tau A=R$, i. e., $A=\tau^{-1} R$ (if $\tau$ is not invertible, then with probability 1 your $A$ does not exist). This is of course full rank with probability 1, since the map $\tau$ just multiplies Lebesgue measure by a non-zero constant.

$\endgroup$
1
  • $\begingroup$ Thank you Fedor! That's exactly what I was looking for. $\endgroup$ Commented Jul 17 at 19:38

You must log in to answer this question.

Start asking to get answers

Find the answer to your question by asking.

Ask question

Explore related questions

See similar questions with these tags.